UTNianos

Versión completa: matematica superior - final 11/02/16
Actualmente estas viendo una versión simplificada de nuestro contenido. Ver la versión completa con el formato correcto.
Páginas: 1 2
Hola! les dejo el final de superior que se tomo hoy.
algunas de las respuestas que me acuerdos son
1a) b=4 y k=13
1b)\[\frac{1}{17}e^{-t}-\frac{1}{17}\cos 4t +\frac{1}{17}\sin 4t\]

2a) k=2
2b) todos reales por ser par.

4) habia que hacer dos iteraciones partiendo desde x= -1

5a) era h= 0.5 y h=0.375

6) no existe k.
buenasssssss me saque 5 no lo puedo creerrrrrrrr, Las respuestas me dieron como vos!
Dejo alguna nota mas: y si me voy acordando pongo mas.
En el punto 2)a) al sacar "a" sub cero para hallar el valor medio (dado que ValorMedio=A0/2 ) habia que multiplicar LAS DOS partes de la integral por 2 . La duda viene porque di AM2 hace 45 mil años y ya no estaba tan canchero con las integrales ajaj pero bueno me avive y me dio K=2. Si no multiplicabas la segunda integral por dos K te daba 4 (que tabine era un valor posible!). Entonces me jugue por multiplicar por dos la integral y acotar el intervalo de la integral tal y como se hacia en los ejercicios.

saludos!
Ayer fui a rendir, me doy cuenta de que estuve muy al limite, me pusieron 4 y creo que me regalaron un poco...
el punto 3 de la transformada Z me dio: x(n)= -2*n * 2^n
me dio así haciéndolo por convolución ya que la X(z) = -2 Z^2 / (z-1) (z+1) (z-2)

la convolución la uso separando las funciones así:

-2 * (Z/(Z^2 -1)) * (Z/(Z-2))

Está bien así??
Gente el 7 de ED lo hicieron con 5 pasos h=0,2 y les dio 2,406610...

No me doy cuenta cómo podría comprobarlo el resultado... gracias
me parece que la respuesta de naatti del punto 5 a es incorrecta ya que para utilizar simpson el intervalo se debe subdividir en subintervalos pares por lo tanto si el intervalo dado que es de [0,1] solo se pueden utilizar los valores de h = 0,2 (genera 4 subintervalos) o bien h = 0,5 (genera 2 intervalos)

los valores de la integral por simpson son para h = 0,2 : As = 1,325
para h = 0,5 : As = 1,491733119

por ultimo, gracias naatti por aportar el examen!!! thumbup3
(17-02-2016 20:26)jmctango escribió: [ -> ]me parece que la respuesta de naatti del punto 5 a es incorrecta ya que para utilizar simpson el intervalo se debe subdividir en subintervalos pares por lo tanto si el intervalo dado que es de [0,1] solo se pueden utilizar los valores de h = 0,2 (genera 4 subintervalos) o bien h = 0,5 (genera 2 intervalos)

los valores de la integral por simpson son para h = 0,2 : As = 1,325
para h = 0,5 : As = 1,491733119

por ultimo, gracias naatti por aportar el examen!!! thumbup3

El intervalo es (0,3) no se ve muy bien pero era un 3
Hola gracias por el aporte!
Te hago unas consulta el 1)b) no debería dar asi? x(t)= e^(-t)/17 -cos(4t)/17 + sen(4t)/68

a mi me dieron estos valores los ejercicios
El 3) me quedo x[n] = 2.(-1)^n

El 7)
h wi
0..........2
0,2.......2,1
0,4.......2,186956522
0,6.......2,264267446
0,8.......2,334093328
1..........2,397907637
At
Buenas, ¿me podrían iluminar un poco como encarar el primer punto?. Me marea mucho los datos que dan.

Gracias.
En el 2a), porque k=2?

Yo hago:

Ao / 2 = 4 => Ao = 8

Y entonces k me termina dando 4.
kseba
yo lo hice asi:

\[\frac{a_0}{2} = 4 \Rightarrow a_0 = 8 \]
\[a_0 = \frac{1}{3} \left ( \int_{-3}^{3} f(x) dx \right ) = 8\]
\[a_0 = \frac{1}{3} \left ( \int_{-3}^{-2} k dx + \int_{-2}^{2} 5 dx + \int_{2}^{3} k dx\right )\]
\[a_0 = \frac{1}{3} (k+5*4+k) \Rightarrow 8*3 = 20+2k\]
\[4 = 2k \Rightarrow k = 2\]



alguno hizo el 3?

a mi me quedo:

\[x(n) = -3+\frac{8}{3}2^{n}+\frac{1}{3}(-1)^{n}\]
El punto 1b no lo entiendo, no se aplica la derivada 2a de TL??

no necesito f(0) e f´(0) para poder resolverlo?

Gracias.
como te dice que el sistema parte del reposo tanto y(o) como y'(0) son nulas
(23-02-2016 18:09)Jarry escribió: [ -> ]kseba
yo lo hice asi:

\[\frac{a_0}{2} = 4 \Rightarrow a_0 = 8 \]
\[a_0 = \frac{1}{3} \left ( \int_{-3}^{3} f(x) dx \right ) = 8\]
\[a_0 = \frac{1}{3} \left ( \int_{-3}^{-2} k dx + \int_{-2}^{2} 5 dx + \int_{2}^{3} k dx\right )\]
\[a_0 = \frac{1}{3} (k+5*4+k) \Rightarrow 8*3 = 20+2k\]
\[4 = 2k \Rightarrow k = 2\]



alguno hizo el 3?

a mi me quedo:

\[x(n) = -3+\frac{8}{3}2^{n}+\frac{1}{3}(-1)^{n}\]

Tenes razon, no me habia dado cuenta que en la segunda condicion x esta en modulo y por lo tanto quedan 3 integrales. A mi solo me quedaban dos, y por eso me daba distinto. Gracias!
kseba escribió:Tenes razon, no me habia dado cuenta que en la segunda condicion x esta en modulo y por lo tanto quedan 3 integrales. A mi solo me quedaban dos, y por eso me daba distinto. Gracias!
si, a mi en un principio me boleo eso.


che Naatti

me parece que hay un pequeño pifie en tu respuesta del 1b)

yo llegue a esto:

\[Y(s) = -\frac{1}{17}\frac{s}{s^2+4^2}+\frac{1}{17}\frac{1}{s^2+4^2}+\frac{1}{17}\frac{1}{s+1}\]

ahora, para antitransformar el segundo termino necesito un 4 ahi arriba, por lo que hago \[\frac{1}{17}/4\]

entonces ese termino me queda asi:
\[\frac{1}{68}\frac{2}{s^2+4^2}\]
entonces si lo puedo antitransformar como seno[/quote]
Páginas: 1 2
URLs de referencia